Use app×
Join Bloom Tuition
One on One Online Tuition
JEE MAIN 2025 Foundation Course
NEET 2025 Foundation Course
CLASS 12 FOUNDATION COURSE
CLASS 10 FOUNDATION COURSE
CLASS 9 FOUNDATION COURSE
CLASS 8 FOUNDATION COURSE
0 votes
204 views
in Algebra by (53.6k points)

\(\lim\limits_{n \to \infty} \sum\limits_{r =1}^n \frac r{1.3.5.7.9.....(2r +1)}\) is equal to

(a) 1/3

(b) 3/2

(c) 1/2

(d) None of these

Please log in or register to answer this question.

1 Answer

+1 vote
by (53.3k points)

Correct option is (c) 1/2

Welcome to Sarthaks eConnect: A unique platform where students can interact with teachers/experts/students to get solutions to their queries. Students (upto class 10+2) preparing for All Government Exams, CBSE Board Exam, ICSE Board Exam, State Board Exam, JEE (Mains+Advance) and NEET can ask questions from any subject and get quick answers by subject teachers/ experts/mentors/students.

...